2 Block Problems | Friction | JEE Physics | Mohit Sir (IIT KGP) | IIT JEE

Поділитися
Вставка
  • Опубліковано 6 вер 2024
  • Two Block problems in Friction is discussed in detail in this video by Mohir Sir. These problems are asked in JEE Main and JEE Advanced exams.
    🔥 PHD SERIES PLAYLIST - bit.ly/3cQSxPT
    #Friction #2BlockSystem #PhDSeries #Eduniti #JEEMain #IITJEE #JEEAdvanced
    #Physics #IITJEEPhysics
    👀 Links - MUST WATCH!! 👀
    🚀 Master PDF File for Physics PYQs and Concept Videos - bit.ly/2ZKb5Of
    🔥 GOLD Mine Link - bit.ly/2VhOGFF

КОМЕНТАРІ • 615

  • @mohitgoenka99
    @mohitgoenka99  2 роки тому +46

    🔥 *PHD SERIES PLAYLIST* - bit.ly/3cQSxPT
    👀 Links - MUST WATCH!! 👀
    🚀 Master PDF File for Physics PYQs and Concept Videos - bit.ly/2ZKb5Of
    🔥 GOLD Mine Link - bit.ly/2VhOGFF

  • @pranayjalan2714
    @pranayjalan2714 2 роки тому +695

    In your previous videos I just commented you to make a video on two block problems and you made it so quickly sir. Thank you so much. your efforts are highly appreciated.

    • @mohitgoenka99
      @mohitgoenka99  2 роки тому +134

      You are most welcome

    • @naveengupta2862
      @naveengupta2862 2 роки тому +42

      Sir rotation ka revision lec bana do

    • @rexyyedits5371
      @rexyyedits5371 2 роки тому +31

      @@naveengupta2862 Tera kaam bhi done Hogaya bro 👌🔥

    • @aryanabhilesh11
      @aryanabhilesh11 2 роки тому +2

      dude can u say the full form of this PHD series

    • @siddharthrai7i
      @siddharthrai7i 2 роки тому +14

      @@aryanabhilesh11 PHoDu series

  • @User75880
    @User75880 7 місяців тому +51

    legends never comment in any educational platform. they just study, like, subscribe and giving millions and billions of love and thanks from their heart....❤

    • @samairamalik3404
      @samairamalik3404 5 місяців тому

      I have a q. in bth cases,5N also applies a downward normal force on 10kg right? we just ignore it bc its in vertical equi?

    • @zero0bb
      @zero0bb 5 місяців тому +1

      ​@@samairamalik3404No, gravitational force acts downwards. To understand normal force imagine you are pushing a wall, the force you give while pushing a wall is defined, we can call it F(human) signifying a human is giving a pushing force. But, the wall will not move, why? because of a multitude of factors and forces, there are too many to take into account but we know that the normal force always manages to equate the force we give on the wall. That is what normal force is, when a rigid body is immovable a normal force is said to act opposite to it. Hope i helped

    • @samairamalik3404
      @samairamalik3404 3 місяці тому +1

      @@zero0bb you did help!! thanks

    • @KalpanaLodha-mg4oy
      @KalpanaLodha-mg4oy Місяць тому +6

      You are not a legend then.

    • @demiyo8424
      @demiyo8424 Місяць тому

      ​@@zero0bbLol,that's wrong.

  • @bluntchar
    @bluntchar Рік тому +30

    I missed the lecture of two block friction in my coaching centre and was struggling until through his past students I came to know about him, extremely grateful for the work he is doing for JEE aspirants. Explanantion was good and now I have some confidence to go through module questions and DPP's.

  • @spyclutch6855
    @spyclutch6855 Рік тому +36

    I am dropper student ... I hate physics since I started preparing for jee ... And I advanced physics ke kaaran hi mera selection nhi ho paya and abhi bhi muje physics itni pasand nhi thi bas jabarjasti karta tha par abhi 1 mahine pehele aapka channel Mila and u made it very easy and I started loving physics and I am now top scorer in physics in my mocks .. thank you so much sir ❤❤❤..love u sir

  • @vishnugautam320
    @vishnugautam320 2 роки тому +76

    ONLY ONE THING.....
    This is one of the best PHYSICS CHANNEL IN INDIA❤️❤️❤️❤️
    This channel is going to have millions of subscribers...

  • @aspirant_IIT-KGP
    @aspirant_IIT-KGP 2 роки тому +35

    Note for my future reference -
    First step, second step for friction ques uk
    We have 2 categories
    Jaha F neeche Wale par lage
    Jaha F upar Wale par lage
    For no. 1
    F max corresponds to f limiting for upar wala
    Final step me F = m sys (a max) [that chotu upar wala box can tolerate)
    When F exceeds F max neeche wala aur upar wale ka FBD alg-alg banake solve Karo! Ese me koi confusion hoga hi ni! Ki neeche Wale box ki final Force ea me m sys hoga ki m box etc etc
    For no. 2
    F max corresponds to f limiting of lower box
    Here don't u dare forget ki f static is self adjusting so acc to values in ques they can ask F min and F max both!..
    F max corresponds to f limiting
    F min corresponds to f kinetic (for lower box)
    Baki all well!..😁👍

  • @drafercuber-shorts8127
    @drafercuber-shorts8127 2 роки тому +69

    This 2 block system is my favorite because my teacher didnt teach us this concept but some how after fighting 1hr i learned this concept by my own

  • @nikhilpratapsingh340
    @nikhilpratapsingh340 2 роки тому +22

    Awesome sir great explanation, i found second approch of these questions and now i feel more confident on this topic. Great Content and great efforts sir.

  • @uttam2274
    @uttam2274 Рік тому +61

    You made this concept crystal clear to me.
    One of the best Physics teacher i have ever found....👍

  • @theeditor-ams3608
    @theeditor-ams3608 14 днів тому +3

    Again , aaj topic ko revise kiya , aaj phir bolunga iss topic pe isssse achha video kisi ne banaya nahi hai ..
    Thank you so much sir ❤

  • @pradakshinamishra6728
    @pradakshinamishra6728 4 місяці тому +5

    My teacher made this concept so tough like it was something out of this planet 😞😭 😭😑😑😑 then i xame here and sir you made it soo simple to understand and made us feel the concept ❤❤❤❤❤❤❤ thank you sir ... # learn like never before is so true 😉

  • @infinity0069
    @infinity0069 5 місяців тому +13

    Got 95 marks in jan attempt...
    Now targeting for 100 marks in April ❤❤
    Thank you so much sir 🙏

  • @praveenraj1656
    @praveenraj1656 Рік тому +2

    At 17:00 when applying the eqn
    We r applying for lower block
    Then i guess there total mass shd be 10+5 =15 not only 10
    Plz correct me if im going wrong anywhere

    • @mohitgoenka99
      @mohitgoenka99  Рік тому

      25-15 =10a equation is just written for lower block dear. I made FBD of both block separately.
      So here mass be 10 kg only.

    • @praveenraj1656
      @praveenraj1656 Рік тому

      @@mohitgoenka99 thank u so much sir🙏👍

  • @poojanjain6069
    @poojanjain6069 2 роки тому +12

    Am I the only one who, first likes the video then watch it🤗
    (Edit-bring more videos of concept which are being repeated)

  • @lnewf2331
    @lnewf2331 9 місяців тому +3

    Never understood two block system in 11th now in 12th finally understanding it Thanks sirr...🙏🙏

  • @runnerboys9277
    @runnerboys9277 Рік тому +3

    my concept is crystal clear sir.
    now can easliy find direction of friction from the miniature visualization u did. in the start

  • @raviShankar-ff8lw
    @raviShankar-ff8lw Рік тому +2

    Please make a video on direction of friction and some time Norma force is MG cos theta and friction question on incline plane and force vertical on friction question

  • @Krishnanjan_Sil
    @Krishnanjan_Sil 4 місяці тому +1

    9:17 Sir, I think that the acceleration on the orange/ yellow block in the free body diagram will be on the left direction as it is pseudo acceleration.

  • @Aravindh.9
    @Aravindh.9 5 місяців тому +1

    5:50 Fmax = 75N,
    fmax = fl = 25N & acc of the system = 5 m/s^2
    Even if the applied force is greater than 75N, Fl = 25N.

  • @fompypompyy535
    @fompypompyy535 2 роки тому +18

    Thank you for uploading this video sir.. was looking forward to your mechanics concepts 😁

  • @jagritisingh2859
    @jagritisingh2859 Рік тому +1

    Sir please make a video on that ques in which 2 blocks are connected with string and we have to find that there will arise tension or not??

  • @samairamalik3404
    @samairamalik3404 5 місяців тому +2

    I have a q. in bth cases,5N also applies a downward normal force on 10kg right? we just ignore it bc its in vertical equi?

  • @shriniwashase
    @shriniwashase 2 роки тому +9

    I just want to send a hundred thousand thank you's to this man ❤ Love you sir and Tq so much boht help hua isse ☺ Please Phd series mai aur bhi imp topics karwa dena

    • @mohitgoenka99
      @mohitgoenka99  2 роки тому +3

      Sure dear. Check the playlist to find many topics

  • @sanskargoantiya3112
    @sanskargoantiya3112 9 місяців тому +3

    Sir es session ke note nhi hai kya by the way great explaination sir your are the gem of physics and also god love you sir❤❤

    • @mohitgoenka99
      @mohitgoenka99  9 місяців тому

      sorry dear, iske pdf notes nahi hai
      keep solving & learning
      & welcome to the era of true learning
      #learnlikeneverbefore

  • @Anupamchess
    @Anupamchess 4 місяці тому +3

    I dont know whether it will come to neet But I will definitely say this is the BESTEST video about 2 block problems... on YT
    just wow!!

  • @KritikaYadav-kd1mf
    @KritikaYadav-kd1mf 26 днів тому +1

    Thanku sir just bcz of u i got 97 percentile in physics 🙏

  • @vineetsingh6369
    @vineetsingh6369 2 роки тому +6

    Great explanation sir...-"friction a necessary evil."

  • @syedarshil5970
    @syedarshil5970 Рік тому +5

    Sir thank you so much for this amazing session it cleared my whole concept about friction between two blocks 💖😊

  • @BLINK-xw3do
    @BLINK-xw3do Рік тому +3

    always has doubt in when it will move together and when not and the way you explained is absolutely incredible I got it all thankyou so much

  • @JARH_ADARSH
    @JARH_ADARSH 7 місяців тому +4

    sir maine abhi mock m two block system k ques m glti ki thi to m yha aagya mistake sudharne thank uh sir ab aage +4 always ...

    • @samairamalik3404
      @samairamalik3404 5 місяців тому

      I have a q. in bth cases,5N also applies a downward normal force on 10kg right? we just ignore it bc its in vertical equi?

  • @Bhavya657-h7o
    @Bhavya657-h7o Місяць тому

    After watching your videos I gain confidence and pyqs bhi easily solve ho jaate h, thanks a ton 🙏🏻

  • @hardikarora8662
    @hardikarora8662 9 місяців тому +2

    greatEST EXPLANATION of two block system ever !!

    • @mohitgoenka99
      @mohitgoenka99  9 місяців тому

      well done dear
      keep solving & learning
      #learnlikeneverbefore

  • @raviShankar-ff8lw
    @raviShankar-ff8lw Рік тому +2

    Please make a video on how to solve HC verma concepts of physics 🙏

  • @Nishantk10
    @Nishantk10 2 роки тому +3

    sir kya mast padhate ho aap !!! charan sparsh!!

  • @user-wd9nu3ih5n
    @user-wd9nu3ih5n 10 місяців тому +5

    Best physics teacher ❤ great sir.

    • @mohitgoenka99
      @mohitgoenka99  10 місяців тому +1

      keep learning dear
      #learnlikeneverbefore

    • @samairamalik3404
      @samairamalik3404 5 місяців тому

      I have a q. in bth cases,5N also applies a downward normal force on 10kg right? we just ignore it bc its in vertical equi?

  • @sumeetsharma8089
    @sumeetsharma8089 2 роки тому +2

    pure gold....india owes you

  • @nishant6758
    @nishant6758 Рік тому +1

    When force is applied on both blocks then it becomes tough, pls make a vdo on this topic

  • @user-jp2wt9gz2t
    @user-jp2wt9gz2t Рік тому +1

    Sir please make video for 3 and 4 block problems for jee advanced 2023

  • @luckypathak6830
    @luckypathak6830 2 роки тому +3

    Sir from the bottom of my heart I literally say thank you for clearing this concept as it scratched my head to understand.
    But now I understood the approach to solve such problems only because of you 🙏🙏🙏
    thank you so much sir, ❤🙏🙏🙏

  • @sumitgupta310
    @sumitgupta310 2 роки тому +3

    Thanks a lot sir bht prblms arhi thi friction m aj clr hogya very nice explanation ❤️🙌🔥

  • @ESPAREAOFrv
    @ESPAREAOFrv Рік тому +6

    after ALAK SIR YOU MADE THE REVOLUTION IN PHYSICS ❤❤❤❤❤❤❤❤❤❤❤❤❤❤❤❤❤❤❤❤

  • @TheAishupramod
    @TheAishupramod 7 місяців тому +3

    Thank you so much sir for making this concept crystal clear!!! Love from Kerala❤

  • @yashjha2661
    @yashjha2661 2 роки тому +2

    Sir please make this type of video for Atwood machine also

  • @saurabh226
    @saurabh226 2 роки тому +2

    Thanku sir
    Sir next nlm ki v concept video

  • @anshukiran8572
    @anshukiran8572 Місяць тому

    Finally understood now I m feeling calm . thankyou sir

  • @straww_8
    @straww_8 Рік тому +6

    13:39

  • @Sona-t8p
    @Sona-t8p Місяць тому

    all set sir love u im a dropper i scored 97 tile in physics in my mains first attempt but still had doubts now i might even score 100tile thank you very much!!!

  • @mr_d3f4ult
    @mr_d3f4ult 2 роки тому +5

    THANK YOU SIR FOR THIS AMAZING PHD SERIES SACH ME PHD KRA DI AAPNE CONCEPT KI........

  • @dkadmicarts1779
    @dkadmicarts1779 7 місяців тому +3

    thanku so much sir ❤❤

  • @aryajawed1576
    @aryajawed1576 2 роки тому +4

    Sir please start a series that covers important concepts through question for important chapters please sir

  • @idiotknowledge3431
    @idiotknowledge3431 2 роки тому +2

    Sir jiiii.....kamaal karte ho sir ji aap. Thank u soooo much sir

  • @rutu3316
    @rutu3316 2 роки тому +3

    thank you so much sir. hats of to all your efforts.

  • @aayushisharma1878
    @aayushisharma1878 2 роки тому +4

    You made it crystal clear. Thank you sir 😊✨.

  • @madhavagrawal5560
    @madhavagrawal5560 Рік тому +1

    sir only becuse of you i am feeling confident in physics even after studying for 1 year from pw i am not feeling that confident but after founding your chanel 1 month before my physics had become so much strong

    • @dharmakurmi1257
      @dharmakurmi1257 Рік тому

      Bhai apne kaise use kia sir ka channel thoda briefly describe krna? 🙏❤

    • @dextergaming5705
      @dextergaming5705 Рік тому

      Same condition bro thank God I found this channel

    • @dextergaming5705
      @dextergaming5705 Рік тому

      ​@@dharmakurmi1257 bro sir ka 20 days challenge follow Kiya hai Maine pyqs lagaye hai 2022 or 2023 or kuch chapters m 2021 ke bhi

  • @vinayjanu7373
    @vinayjanu7373 Рік тому +2

    So nice we need teacher like him

  • @asitpurohit_108
    @asitpurohit_108 17 днів тому

    Static friction should be always equal to applied force then how come it’s coming F/3 in the above case , and why would we always start with assumption that it will have same acceleration .
    Can u answer this

  • @tushanpriya5784
    @tushanpriya5784 Рік тому +2

    Most Valuable channel for JEE ASPIRANTS

    • @samairamalik3404
      @samairamalik3404 5 місяців тому

      I have a q. in bth cases,5N also applies a downward normal force on 10kg right? we just ignore it bc its in vertical equi?

  • @satyam99999
    @satyam99999 Рік тому +4

    In the last case The max friction on 5kg BLOCK is 25N HOW IT WILL JUST MOVE AT 15N
    Btw IT WAS AN AWESOME SESSION
    LOVED IT❤❤❤❤❤
    Thank You So Much Sir🙏🙏🙏🙏🙏

    • @mohitgoenka99
      @mohitgoenka99  Рік тому

      Keep it up

    • @subhomandal6549
      @subhomandal6549 Рік тому +1

      same question

    • @a_hi9856
      @a_hi9856 8 місяців тому +1

      bro try to understand, only one block is not moving, the whole system is moving.... (the moment F becomes larger than fmax i.e 25, both blocks will then start moving at seperate acceleration)

  • @girishsahu4817
    @girishsahu4817 2 роки тому +2

    Sir ji hame quality education dene aur visualise krke padhane ke liye thanx..🙏

  • @user-fg1ii8wc3x
    @user-fg1ii8wc3x 5 місяців тому +1

    this channel is soooo underrated ,I really like the phd series they were very useful thank u so much sirr...

  • @balak2212
    @balak2212 8 місяців тому

    Sir agar question me kinetic or static dono friction alag alag hai toh phir video ke sabse akhiri wala case me kinetic lenge ha static?

  • @Aravindh.9
    @Aravindh.9 5 місяців тому

    16:21 acc of the system = 1 m/s^2, Fmax = 30N.
    Range of applied force: 15 - 30N.

  • @anand3071
    @anand3071 Рік тому +2

    Best content of physics in whole youtube ❤️❤️❤️❤️

  • @udayancarromgaming5907
    @udayancarromgaming5907 27 днів тому

    Sir last problem mein range 15 to 30 ki jagah par 25 to 30 hoga
    Kyuki 5Kg par agar ham 16N external lagaye toh f1 use balance kar dega kyuki uska max limit 25N hai par neeche wale 10Kg pe toh friction 16 N he lagega aur uspe friction 15N he laga toh uske pass acceleration a gaya but upar wala rest pe hai.
    Please let me know If I am correct or wrong

  • @yashwantsawant7736
    @yashwantsawant7736 Рік тому +1

    My physics teacher tought us 3 block system i understand it but they donot tought us 2 block system
    Thankyou sir for your videos
    Jee 2024 aspirant
    Target 220

  • @jayeshshrivastava1649
    @jayeshshrivastava1649 2 роки тому +1

    dil se thankyou sir ye concept ekdum chamakgaya hai bahut problem hoti...in dealing these type of questions... thankyou

  • @hemlata29875
    @hemlata29875 2 роки тому +2

    Thanks sir for this video bcz this concept become more strong only of u ....

  • @ayushlad1520
    @ayushlad1520 2 роки тому +3

    Sir please make such videos which will explain important topics in short.... Just like projectile motion....It will be really helpful... Thank u for this one🤩😇😇

  • @hanishagarwal8715
    @hanishagarwal8715 2 роки тому +1

    Is type mai to bhut problem definite aati thi pahle but sir ab ye mera strength ban gya
    Thanku sir 🙏 ☺

  • @PriyanshuDhami-ch1sj
    @PriyanshuDhami-ch1sj 2 роки тому +10

    Thank you sir for such a good explanation ❤️😊
    Sir plz keep on bringing these type of important concept videos sir🙏🏻🙏🏻

  • @sameersamal1708
    @sameersamal1708 8 місяців тому +5

    if i wll get iit i will surely tell in my interview that mr rajwant singh and mr mohit goenka are gems of physics

    • @mohitgoenka99
      @mohitgoenka99  8 місяців тому

      lovely dear
      keep solving, learning and revising
      #learnlikeneverbefore

    • @law3583
      @law3583 8 місяців тому

      atleast he has some dreams unlike u@@comradesofficialff8800

    • @AyushAwasthi_JEE2025
      @AyushAwasthi_JEE2025 Місяць тому

      mili IIT?

    • @sameersamal1708
      @sameersamal1708 Місяць тому

      @@AyushAwasthi_JEE2025 iit ism dhanbad

    • @AyushAwasthi_JEE2025
      @AyushAwasthi_JEE2025 Місяць тому

      @@sameersamal1708 ohh congrats bro...btw rank?

  • @AnujParmar-hw2tu
    @AnujParmar-hw2tu Рік тому +3

    Super explanation sir❤

  • @LadderVictims
    @LadderVictims 2 роки тому +6

    No notes for this lecture :(

  • @LieutenantGeneral
    @LieutenantGeneral 2 роки тому +2

    I am in love with this channel great content. Also is it possible to increase the size of that red pointer it will be helpful plz

  • @harshdevda3529
    @harshdevda3529 2 роки тому +2

    I'm from pw batch and friction was not clear i had doubts thanks sir 🔥 now I understood everything

    • @a.m.videos3254
      @a.m.videos3254 Рік тому

      No offense...but RJ sir is really overrated

  • @Andriano.
    @Andriano. Рік тому +4

    Sir this lecture was awesome and gajbestic ❤️❤️💗❤️💗❤️💗❤️💗
    And one request 🔥🔥🔥
    Please make a PhD series on power questions including on water motor's power please sir 🙏🏻🙏🏻🙏🏻🙏🏻

  • @NikhilBansal691
    @NikhilBansal691 Рік тому

    why should at 19.51 it move with constant accaleration when f is greater than 15 for eg if i take F as 16 N the f1 is 16 N which balances it out and the accaleration of lower block block becomes 0.1m/s which clearly shows a will not be constant

  • @user-dn7yf7cs4l
    @user-dn7yf7cs4l Місяць тому +1

    The most of all videos on two block systems I have ever watched is not so good as you taught in too depth from basic to advance. Thank you sir ❤❤❤❤

  • @AshishJhq
    @AshishJhq Рік тому +3

    Crystal clear concept 🚀🚀

  • @JAGMEETHEBEST
    @JAGMEETHEBEST 2 роки тому +7

    I WILL COME BACK TO WATCH THIS VIDEO IN 2HRS !!🔥🤟

  • @tejendras98
    @tejendras98 Рік тому +7

    You deserve many many many more subs sir and one day with this dedication ik you will become the greatest physics teacher on this planet

  • @ananyajain4219
    @ananyajain4219 2 роки тому +1

    Best channel for physics on youtube👍🏻👍🏻👍🏻

  • @mohitahuja9412
    @mohitahuja9412 Рік тому +1

    At 20:14 what if I applies a force of 27N f1m(maximum static friction is 25N) therefore making fbd for the case we get 2N as the net forward driving force with acc=0.4 m/s²
    But that same 25N (frictional force) would be acting on 10kg block also and the resisting force there equals 15N and hence as a result 10N is the net force.. therefore acceleration should be 1m/s²
    Then sir how can the range be 15 to 30N.. correct me if I am wrong anywhere

  • @Adobeaccount-jk9eg
    @Adobeaccount-jk9eg 5 місяців тому +2

    Done ✅
    Day 2/7 phy hard challenge

  • @Pulkit-tx6ih
    @Pulkit-tx6ih Рік тому +2

    Sir this was very relevant and helpful.
    Enjoyed a lot and thank you for everything you did to us .
    Because yof you I liked this concept 😀

  • @Palakthakur9605
    @Palakthakur9605 10 місяців тому +2

    Crystal clear 2 block system problems 😊thnks a lot of sir❤❤

    • @mohitgoenka99
      @mohitgoenka99  10 місяців тому +1

      well done dear
      keep learning

    • @Palakthakur9605
      @Palakthakur9605 10 місяців тому

      @@mohitgoenka99 sir Aaj Mera test tha advance ka mere se NLM ke spring k concept k ques nhi bnn rhe the mere se kchh tips bta dejia kaise krun..??

  • @surajmundhe2489
    @surajmundhe2489 2 роки тому +7

    Your concepts are just lit 🔥, finally found this channel 🔥❤

    • @mohitgoenka99
      @mohitgoenka99  2 роки тому +3

      Thanx dear & welcome to Eduniti

    • @surajmundhe2489
      @surajmundhe2489 2 роки тому +2

      @@mohitgoenka99 Thanks a lot sir, ye topic abhi tak clear nahi hua tha, ab sab clear hai 🔥, btw sir I am neet aspirant to neet ke liye koi separate playlist hai kya? 🤔

    • @mohitgoenka99
      @mohitgoenka99  2 роки тому +3

      @@surajmundhe2489 No dear but our PhD and revision series are equally relevant for you all

    • @surajmundhe2489
      @surajmundhe2489 2 роки тому

      @@mohitgoenka99 okk sir ❤I definitely follow them, then 🔥✌

    • @mohta.
      @mohta. 8 місяців тому

      @@surajmundhe2489 bhai kaisa rha neet aapka

  • @ashikagupta1503
    @ashikagupta1503 Рік тому +1

    Sir aapka explanation to itna achcha hai ki iit phodne ka confidence agya hai thank you so much sir for such a wonderfull content

  • @harshrajchoudhary4009
    @harshrajchoudhary4009 2 роки тому +1

    Bhaiya in the last case while drawing the FBD of 10kg block you took 10kg as an independent block and not as a system.because u write the EQ as.
    25-15=10a
    Here you have only taken the mass of 10kg block and multipled it with 'a' But in same EQ you have taken f2 = 15N
    Were the value 15N has come from the contribution of both the blocks weight.
    Why can't we take the value of f2 as 10N only.
    Bhaiya Plz explain 😥🙏🙏🙏

  • @nishantsadawarte3886
    @nishantsadawarte3886 8 місяців тому +1

    Sir friction naso me bas gaya hai lecture dekh ke, maza aa gaya❤❤

  • @sudhirkurude4031
    @sudhirkurude4031 2 роки тому +3

    Oh nice you cleared my doubt thank u so much ❤️

  • @aryangodara8538
    @aryangodara8538 Рік тому +2

    Really sir loved it a lottttt😍😍

  • @AP__2004
    @AP__2004 Рік тому +1

    Sir plzz iss lecture ka pdf available kar dijiye .!!🙏🙏

  • @crazyman_1810
    @crazyman_1810 Рік тому +1

    Sir I know where students face problems thanks sir

  • @meharunnisa5861
    @meharunnisa5861 2 роки тому +1

    This is the most underrated channel and you are a star sir ..thank you very much for this beautiful class sir 🙏🙏

  • @InfinusPlayZz
    @InfinusPlayZz 5 місяців тому +1

    Currently Preparing for BITSAT-2024 sir, these videos are still extremely helpful!

  • @kalpana6468
    @kalpana6468 2 роки тому +1

    Sir in this in the 1st case when force was applied on the lower block you didn't subtracted the friction due to 5kg block on the 10 kg but in the 2nd case you did can you please explain this thing , thank you

  • @Dharun-ge2fo
    @Dharun-ge2fo Місяць тому +1

    Why didn't we write consider the two blocks as a system and find Fmax =(15)(Acommon)

  • @allenstudent4399
    @allenstudent4399 Рік тому +2

    Going on way completion of syllabus and this marvelous series I sway helping a lot ♥️♥️♥️♥️♥️♥️❤️❤️❤️❤️❤️❤️❤️💕💕💕